ไม่สามารถเล่นวิดีโอนี้
ขออภัยในความไม่สะดวก

Ecuación de la Olimpiada de Matemática

แชร์
ฝัง
  • เผยแพร่เมื่อ 31 พ.ค. 2024
  • Olimpiada de Matemática
    Link de la lista de reproducción de cálculo diferencial: • Cálculo Diferencial
    Link de la lista de reproducción de cálculo integral: • Cálculo Integral
    Link de la lista de reproducción de ecuaciones diferenciales: • Ecuaciones diferenciales
    Hola, si quieres invitarme un café te dejo el siguiente enlace: paypal.me/ichigoomath?locale....

ความคิดเห็น • 162

  • @rauljordan11
    @rauljordan11 หลายเดือนก่อน +180

    Es muy fácil si pasas el número 1984 a binario que es 11111000000 entonces es la resta de 100000000000 menos 1000000. Es decir m=11 y n=6.

    • @IchigooMatematicas
      @IchigooMatematicas  หลายเดือนก่อน +41

      Hola, excelente desarrollo, no lo había visto de esa manera.

    • @tomasbarrancocorreas5028
      @tomasbarrancocorreas5028 หลายเดือนก่อน +9

      Sublime

    • @albertoloza7259
      @albertoloza7259 หลายเดือนก่อน +11

      Genio

    • @ksaver3003
      @ksaver3003 หลายเดือนก่อน +11

      Entre a ver por ocio, y me impulsaste a aprender binario para entender su forma de resolverlo.

    • @hugo_tbbto3552
      @hugo_tbbto3552 หลายเดือนก่อน +2

      Hola amigo, no termino de comprender tu forma de resolverlo, recién aprendí lo básico de los sistemas numéricos (b,o,h), me podrías dar una breve explicación si no es mucho pedir

  • @galojara3207
    @galojara3207 27 วันที่ผ่านมา +2

    Muy buena explicación. El hecho de intuir que m>n y utilizar el cambio de variable fue muy bueno.

  • @dugarmendez
    @dugarmendez หลายเดือนก่อน +4

    Excelente explicación. Muy didáctica y útil. Saludos cordiales y gracias.

  • @sarayanez2469
    @sarayanez2469 หลายเดือนก่อน +2

    Estoy impresionada maestro...reciba mis respetos y cordiales saludos.

    • @IchigooMatematicas
      @IchigooMatematicas  หลายเดือนก่อน +1

      Hola, buen día, le agradezco mucho su apoyo y comentario 😃.

  • @alejandrohenaopatino3893
    @alejandrohenaopatino3893 หลายเดือนก่อน +6

    Muy lindo ejercicio

    • @IchigooMatematicas
      @IchigooMatematicas  หลายเดือนก่อน +4

      Hola, muchas gracias por su apoyo y comentario 😃.

  • @migueluzi
    @migueluzi หลายเดือนก่อน +4

    Genial maestro, con gran enfasis en el álgebra.

    • @IchigooMatematicas
      @IchigooMatematicas  หลายเดือนก่อน +1

      Hola, muchas gracias por su apoyo y comentario 😃.
      Hay varios métodos de resolver y aquí mostré uno de ellos.

  • @pelayomedina2174
    @pelayomedina2174 หลายเดือนก่อน +6

    Podemos expresarlo como 2^n*(2^(m-n) -1)
    Si factorizamos 1984 nos damos cuenta de que es 2^6 * 31 que a su vez es 2^6*(2^5 -1)
    Luego n=6 y m-n=5 luego m=11

    • @IchigooMatematicas
      @IchigooMatematicas  หลายเดือนก่อน +1

      Hola, excelente razonamiento.
      Le agradezco mucho su apoyo y comentario 😃.

  • @ChrisF01zx
    @ChrisF01zx หลายเดือนก่อน +5

    Que seco..muy buen análisis 👏🏻

    • @IchigooMatematicas
      @IchigooMatematicas  หลายเดือนก่อน +2

      Saludos, muchas gracias por su apoyo y comentario 😃☺️.

  • @arielespinoza8695
    @arielespinoza8695 26 วันที่ผ่านมา +2

    Muchas gracias profesor, me gustó mucho su explicación :)

    • @IchigooMatematicas
      @IchigooMatematicas  26 วันที่ผ่านมา +1

      ¡Hola! Muchas gracias por su apoyo y comentario 😊.

  • @Daniel-wg9pv
    @Daniel-wg9pv หลายเดือนก่อน +2

    Excelente explicacion maestro, lo he visto muy fácil de entender como estudiante ❤

    • @IchigooMatematicas
      @IchigooMatematicas  หลายเดือนก่อน

      Excelente, lo felicito.
      Muchas gracias por su apoyo y comentario 😃.

  • @eustaquiomora6648
    @eustaquiomora6648 หลายเดือนก่อน +1

    Gracias Maestro, Dios le bendiga.

    • @IchigooMatematicas
      @IchigooMatematicas  หลายเดือนก่อน +1

      Hola, muchas gracias por su apoyo y comentario.
      Igualmente que Dios lo bendiga.

  • @pepegarcia6348
    @pepegarcia6348 หลายเดือนก่อน +11

    Con los valores obtenidos se cumple la igualdad, pero no ha demostrado que haya una única solución. De hecho hay otras soluciones válidas.

    • @IchigooMatematicas
      @IchigooMatematicas  หลายเดือนก่อน +4

      Hola, considero que demostrar que haya cierta cantidad de soluciones es mucho más complejo que encontrar algunas soluciones.
      Saludos

    • @pepegarcia6348
      @pepegarcia6348 หลายเดือนก่อน +1

      @@IchigooMatematicas Me refiero a que habría que especificarlo en la respuesta. Por ejemplo, si te dan la ecuación y=2x, no basta con responder y=2, x=1. Con esos valores se cumple la igualdad, pero la solución no es correcta, o al menos no es completa, porque la igualdad se cumple para cualquier valor de y que sea el doble que x.

    • @bltnbros122
      @bltnbros122 หลายเดือนก่อน +1

      ​@@IchigooMatematicasClaro, pero de eso se trata resolver un problema. En todo caso si el enunciado fuese ''encuentre una solución'' entonces ahi si seria valido.

    • @IchigooMatematicas
      @IchigooMatematicas  หลายเดือนก่อน +2

      @@bltnbros122 es verdad.

  • @serafinbenitez8166
    @serafinbenitez8166 หลายเดือนก่อน +5

    Me interesó más el análisis para los principiantes! Muy bien!

  • @voucomentaroquepenso.logo7312
    @voucomentaroquepenso.logo7312 หลายเดือนก่อน +3

    Saludos desde Brasil. Essa resolução foi sensacional. 👌🏻 Parabéns professor 👏🏻👏🏻

    • @IchigooMatematicas
      @IchigooMatematicas  หลายเดือนก่อน +2

      Hola, gracias por su apoyo y comentario 😃.
      Saludos a toda la gente del bonito Brasil.

  • @angellorenzorubio9439
    @angellorenzorubio9439 หลายเดือนก่อน +2

    Excelente demostración felicitaciones...!!!😊😊

    • @IchigooMatematicas
      @IchigooMatematicas  หลายเดือนก่อน +1

      Saludos, le agradezco mucho su apoyo y comentario 😃.

  • @lorenaalexandrahuamanbueno2322
    @lorenaalexandrahuamanbueno2322 หลายเดือนก่อน +7

    Con factores primos. Hallando la expresión 1984 = 2^6x31 = 2^6x(32-1) = 2^6x(2^5-2^0) = 2^11 - 2^6. Entonces m=11 y n=6.

    • @IchigooMatematicas
      @IchigooMatematicas  หลายเดือนก่อน +2

      Excelente desarrollo.
      Muchas por su apoyo y comentario 😃

  • @maxwellarregui814
    @maxwellarregui814 หลายเดือนก่อน +1

    Buenas tarde estimado amigo Apolo. Gracias este lindo ejercicio. Éxitos.

    • @IchigooMatematicas
      @IchigooMatematicas  หลายเดือนก่อน +2

      Gracias por apoyarme en este canal amigo Maxwell.

  • @cristianpatricio3579
    @cristianpatricio3579 หลายเดือนก่อน +1

    Muchas Gracias 😊

  • @Claudio_R
    @Claudio_R หลายเดือนก่อน +12

    2^m debe ser > que 1984, entonces busco m hasta que 2^m sea mayor que 1984. En este caso para m=11, 2^11=2048. Le restamos 1984 y nos da 64 que es igual a 2^6, por lo tanto m=11 y n=6.

    • @IchigooMatematicas
      @IchigooMatematicas  หลายเดือนก่อน +5

      Excelente desarrollo, le agradezco mucho su apoyo y comentario 😃.

  • @unanbu8915
    @unanbu8915 หลายเดือนก่อน +1

    Impresionante demostración

    • @IchigooMatematicas
      @IchigooMatematicas  หลายเดือนก่อน +1

      Hola, muchas gracias por su apoyo y comentario 😃.

  • @Albertogonzalez-xq2ou
    @Albertogonzalez-xq2ou หลายเดือนก่อน +1

    Lindo ejercicio, tiene su detalle. . Saludos 🇨🇷

    • @IchigooMatematicas
      @IchigooMatematicas  หลายเดือนก่อน +1

      Hola, muchas gracias por su apoyo y comentario 😃.

  • @urify951
    @urify951 หลายเดือนก่อน +18

    Es fácil afirmar que 1984 es igual a "1986 - 2", a "1988 - 4", "1992 - 8", y así seguimos hasta que encontramos que es igual a "2048 - 64". Por lo tanto, los valores de m y n son 11 y 6 respectivamente. No sé si en olimpiadas de matemáticas sea de gran exigencia usar el mismo procedimiento que usted empleó.

    • @IchigooMatematicas
      @IchigooMatematicas  หลายเดือนก่อน +6

      Hola, muchas gracias por su apoyo y comentario.
      Puede ser usado cualquier método, lo importante es la solución.

    • @urify951
      @urify951 หลายเดือนก่อน +2

      @@IchigooMatematicas Muchas gracias, buenas noches.

    • @Dreikcou
      @Dreikcou หลายเดือนก่อน +1

      hola, perdón no lo alcanzar a ver por que el hecho de que 2048-64 implica que m=11 y n=6, ¿sería tan amable de explicarme por favor?

    • @Dreikcou
      @Dreikcou หลายเดือนก่อน +1

      ya vi, aplicaste el logaritmo base dos, que tonto, JAJAJm, gracias de todos modos si es que contestas :)

    • @urify951
      @urify951 หลายเดือนก่อน +1

      @@Dreikcou Hola, claro le explico:
      Lo que nos pide el problema es buscar los exponentes de dos potencias; la diferencia entre esas potencias es 1984; lo que implica que se puede evaluar la suma de 1984 más 0 (1984 + 0), pero la operación se puede expresar también como 1984 más y menos una potencia de base 2 (1984 + 2 - 2, por ejemplo). Lo que hice fue evaluar la suma con cada potencia y al final obtuve el resultado deseado, pero no es la única forma; cómo se ha mostrado, se puede buscar la potencia de base 2 más cercana a 1984, el cuál es 2048 (que es igual a 2^11); al restarlo con 1984 obtendrá 64 (que es igual a 2^6). Al final nos queda que 2^m - 2^n = 2^11 - 2^6; y por simple intuición decimos que m=11 y n=6.

  • @fromant65
    @fromant65 หลายเดือนก่อน +6

    Apenas vi la miniatura pense "mmm, 1984 esta cerca de 2048, y 64 parece cercano...", lo puse en la calculadora y funcionó

    • @IchigooMatematicas
      @IchigooMatematicas  หลายเดือนก่อน +1

      Excelente.
      Muchas gracias por su apoyo 😌 y comentario.

    • @starklosch
      @starklosch หลายเดือนก่อน +1

      Yo también lo pensé así. Sabiendo las potencias de 2 se puede buscar la siguiente potencia mayor a 1984 (2048). Entonces le restas 1984 y obtienes la otra potencia de 2 (64)

    • @fromant65
      @fromant65 หลายเดือนก่อน +1

      @@starklosch de hecho siempre que tengas que x=2^b-2^c, va suceder que 2^(b-1)≤x

  • @brauliodiniz7753
    @brauliodiniz7753 หลายเดือนก่อน +3

    Muy bien

  • @juanc.suarez6360
    @juanc.suarez6360 หลายเดือนก่อน +7

    Desde el inicio se tiene que decir si m y n son enteros o racionales o relaes. Solo ese detalle hace que uno no quiera seguir viendo el vídeo.

    • @FAMILIAJIMENEZ-eg8jj
      @FAMILIAJIMENEZ-eg8jj หลายเดือนก่อน +2

      ES MEDIO SACADO DE LA MANGA EL CAMBIO DE VARIABLE, NO ES CIERTO QUE SE NOTA NADA

    • @IchigooMatematicas
      @IchigooMatematicas  หลายเดือนก่อน +2

      Saludos, es importante hacer lo que le comentó, gracias.

    • @IchigooMatematicas
      @IchigooMatematicas  หลายเดือนก่อน +2

      Saludos.

    • @jesushurtado2780
      @jesushurtado2780 หลายเดือนก่อน +2

      Eso es correcto, debe decir las condiciones de m y n como números naturales enteros y positivos como.premisas condicionantes iniciales

  • @restablex
    @restablex หลายเดือนก่อน +6

    Yo busqué una potencia de 2 q sea mayor o igual a 1984
    2^11 = 2048
    Le restas 1984 y te da 64 que es 2^5.

    • @IchigooMatematicas
      @IchigooMatematicas  หลายเดือนก่อน +3

      Hola, buen razonamiento.
      Muchas gracias por su apoyo y comentario 😃.

  • @sirjuliusdeviscensus114
    @sirjuliusdeviscensus114 12 วันที่ผ่านมา +1

    bacano, gracias,,,,

  • @fulljsu3glitches
    @fulljsu3glitches หลายเดือนก่อน +18

    Yo calculándolo en 1 minuto sabiendo que 2^11=. 2048 y restandole el 1984 para sacar 64 = 2^6 => m = 11 y n=6 soy un genio??

    • @IchigooMatematicas
      @IchigooMatematicas  หลายเดือนก่อน +10

      Hola, buen desarrollo, gracias por su apoyo y comentario.
      Todos somos genios dentro de nuestras capacidades.

  • @gomezdigital7595
    @gomezdigital7595 หลายเดือนก่อน +2

    muy bien

  • @pedrodelreal5596
    @pedrodelreal5596 หลายเดือนก่อน +1

    buen video bro

  • @lupito6185
    @lupito6185 หลายเดือนก่อน +7

    yo pensé en 1984 como 2048 - 64= 2^11 - 2^6, pero es más heurística que cualquier otra cosa, no sabría hacerlo con rigor :( (comentario antes de ver el video)

    • @IchigooMatematicas
      @IchigooMatematicas  หลายเดือนก่อน +2

      Hola, muchas gracias por su apoyo y comentario 😃.

    • @fofiadrian331
      @fofiadrian331 หลายเดือนก่อน +1

      X2

  • @royotech
    @royotech หลายเดือนก่อน +1

    Felicitaciones wey

  • @user-vb9hh6us7s
    @user-vb9hh6us7s หลายเดือนก่อน +10

    Me confunde la parte en que Iguala los factores, porque que pueden igualar? Esa no me la sé, no entiendo porque se puede hacer eso.

    • @IchigooMatematicas
      @IchigooMatematicas  หลายเดือนก่อน +10

      Saludos.
      2^n siempre va a ser par o 1, por ello se iguala al número par.
      Y 2^(k+1) va a ser impar por ellos se iguala con el número impar.
      Muchas gracias por su apoyo y comentario 😃.

    • @ernestoblanco4353
      @ernestoblanco4353 หลายเดือนก่อน +1

      Porque son potencias de 2, y el otro factor es primo

    • @IchigooMatematicas
      @IchigooMatematicas  หลายเดือนก่อน +1

      @@ernestoblanco4353 Es correcto, saludos.

    • @josemarino8787
      @josemarino8787 หลายเดือนก่อน +1

      Capaz que otra forma de verlo es así. Fijate que esa igualdad es equivalente a
      2^(n-6)×(2^k-1)=31.
      Esto te dice que 2^(n-6) es un divisor del número primo 31. Así, debe ser 2^(n-6)=1 o 2^(n-6)=31. La segunda igualdad no es posible, así que debe verificarse la primera. Y esto pasa sii es n=6.

    • @IchigooMatematicas
      @IchigooMatematicas  หลายเดือนก่อน +1

      @@josemarino8787 Excelente.

  • @ddffnn1
    @ddffnn1 หลายเดือนก่อน +6

    Es una ecuación con dos incógnitas. Se puede dejar una variable en función de la otra despejando de la ecuación.
    Resulta una ecuación logarítmica de base 2 con infinitas soluciones.
    Por ejemplo:
    n = 6 ; m = 11
    n = 7 ; m = 11.04439412
    n = 8 ; m = 11.12928302
    n = 9 ; m = 11.28540222
    .
    .
    .
    m = log2(1984+2^n)

  • @gabrielcastrovivas9520
    @gabrielcastrovivas9520 หลายเดือนก่อน +12

    2048 - 64 = 1984.

  • @alonsomaroto9138
    @alonsomaroto9138 หลายเดือนก่อน +2

    Me pareció muy interesante. En las olimpiadas se tiene aue hacer un desarrollo?, solo tengo simple curiosidad. Se me huzo mas facil hacerlo mentalmente que realizando un procedimiento

    • @IchigooMatematicas
      @IchigooMatematicas  หลายเดือนก่อน +1

      Hola, muchas gracias por su apoyo y comentario 😃.
      Cualquier método es válido.

  • @diezjoaquin1570
    @diezjoaquin1570 หลายเดือนก่อน +1

    Puedes pasar el sitio web del que obtubiste el problema? Muy clara explicación! :)

    • @IchigooMatematicas
      @IchigooMatematicas  หลายเดือนก่อน +2

      Hola, muchas gracias por su apoyo y comentario.
      Es del libro Deminovich.

  • @Jose-Kan
    @Jose-Kan หลายเดือนก่อน +5

    Pero m y n son enteros? Porque podria haber mas posibilidades infinitas

    • @IchigooMatematicas
      @IchigooMatematicas  หลายเดือนก่อน +5

      Es correcto deben ser enteros, muchas gracias por su apoyo y comentario.

    • @GUTY1729
      @GUTY1729 หลายเดือนก่อน

      ​@@IchigooMatematicasalguna demostración de que sea la única solución entera?

  • @marcoantonionunezcosinga7828
    @marcoantonionunezcosinga7828 หลายเดือนก่อน +1

    Muy bonito el ejercicio

    • @IchigooMatematicas
      @IchigooMatematicas  หลายเดือนก่อน +1

      Saludos, que bueno que le gustó el video.

  • @joryeo10
    @joryeo10 หลายเดือนก่อน +9

    Faltó decir en el planteamiento del problema que m y n debian ser números enteros porque si no hay infinitas soluciones

    • @IchigooMatematicas
      @IchigooMatematicas  หลายเดือนก่อน +3

      Hola, muchas gracias por su apoyo y comentario.
      Sí me hizo falta poner las restricciones al inicio.

  • @lucianopires1184
    @lucianopires1184 หลายเดือนก่อน +14

    podemos passar a asser el problema en binario y se queda mas facyl:
    10^m -10^n = 11111000000
    podemos ver con facilidad que:
    (=)10^m -10^n= 100000000000 -1000000= 10^1011 -10^110
    entonces podemos inferir que:
    m=1011 e n=110
    passando para base decimal:
    m=1*2^3+0*2^2+1*2^1+1*2^0=11
    n=1*2^2+1*2^1+0*2^0=6
    assi se resuelve el problema como un autista

    • @IchigooMatematicas
      @IchigooMatematicas  หลายเดือนก่อน +5

      WOW, excelente desarrollo.

    • @lucianopires1184
      @lucianopires1184 หลายเดือนก่อน +3

      @@IchigooMatematicas
      Siqueira és reasonable¿?
      Pensé que sería el as-me reir

    • @andycruz3893
      @andycruz3893 หลายเดือนก่อน +3

      🆗 🐚🫵🤱

    • @ChrisF01zx
      @ChrisF01zx หลายเดือนก่อน +2

      😂

    • @lucianopires1184
      @lucianopires1184 หลายเดือนก่อน +1

      @@andycruz3893 xd

  • @maurogadea1067
    @maurogadea1067 หลายเดือนก่อน +2

    Se puede resolver por metodos número ?

    • @IchigooMatematicas
      @IchigooMatematicas  หลายเดือนก่อน +2

      Hola, si, es interesante ese método, lo haré en video.

  • @user-es3jy9id7e
    @user-es3jy9id7e หลายเดือนก่อน +4

    Ese problema es fácil d resolver si simplemente sabes de potencias y sus resultados pero no está de más ver procedimientos de como hacerlo

    • @IchigooMatematicas
      @IchigooMatematicas  หลายเดือนก่อน +3

      Hola, le agradezco mucho su apoyo.
      Es correcto, en ocasiones otros métodos resultan interesantes.

    • @CodriYT
      @CodriYT หลายเดือนก่อน

      Elchiste es aprender como resolver por q los datos de la ecuación pueden cambiar, eso es lo que el profe enseña, ejm= 14^m-1789^n= 17738

  • @cadenaspoke9027
    @cadenaspoke9027 หลายเดือนก่อน +3

    ¿Pero cómo pruebas la unicidad de la solución?

    • @IchigooMatematicas
      @IchigooMatematicas  หลายเดือนก่อน +5

      Hola, le agradezco mucho su apoyo y comentario.
      El objetivo del vídeo no es ese, pero es interesante analizarlo en un próximo video.

    • @cadenaspoke9027
      @cadenaspoke9027 หลายเดือนก่อน +7

      @@IchigooMatematicas Entiendo. Preguntaba porque, obviamente, siendo un problema de Olimpiada no basta con probar que existe una solución, sino hallar todos los valores posibles de m y n, de ahí la dificultad del problema. Por norma general, y este creo que también es el caso (no he echado la cuenta) la solución suele ser única, pero no por ello hay que sobreentender la unicidad una vez encontramos una solución válida. ¡Un saludo!

    • @DebemosSaberSabremos
      @DebemosSaberSabremos หลายเดือนก่อน +1

      creo que el teorema fundamental de la aritmética garantiza la unicidad , desde que 2^k -1 no es divisible por 2

    • @cadenaspoke9027
      @cadenaspoke9027 หลายเดือนก่อน +2

      @@DebemosSaberSabremos Correcto, probablemente esa sea la jugada. 1984 no es potencia de 2, así que 2^k - 1 ≠ 1, y por tanto por el TFAr 1984 admite una factorización única en términos de números primos. Todos los doses caerían sobre el primer factor y todos los primos impares estarían compactificados en 2^k - 1. Y ahora sí, factorizando 1984 obtenemos el resultado del vídeo.

  • @blimao3951
    @blimao3951 หลายเดือนก่อน +5

    (Yo utilizando logarítmos) 🗿

  • @ChrisF01zx
    @ChrisF01zx หลายเดือนก่อน +5

    5:03 Esa regla o ley como se llama?? Porque funciona??

    • @IchigooMatematicas
      @IchigooMatematicas  หลายเดือนก่อน +3

      Hola, 2^n sólo puede ser par, como 2^6 y la otra expresión es impar, por ello se igualan de esa forma.
      Le agradezco mucho su apoyo y comentario 😃.

    • @antoniou.h.1409
      @antoniou.h.1409 หลายเดือนก่อน +2

      A*B=1*2… por lo tanto, A=1 y B=2, es una consideración porque en ambos lados son multiplicaciones.

    • @IchigooMatematicas
      @IchigooMatematicas  หลายเดือนก่อน +2

      @@antoniou.h.1409 sumado a eso uno es par y otro impar.

    • @ChrisF01zx
      @ChrisF01zx หลายเดือนก่อน +1

      Aaaa claro, osea que si digo que 2^n es 31 estaría mal mi análisis, porque la potencia tiene base 2 y siempre da lo mismo el exponente será par el resultado, ya comprendo 😅 gracias

    • @ChrisF01zx
      @ChrisF01zx หลายเดือนก่อน +2

      Osea que no siempre puedo usar ese artilugio, siempre debo analizar la igualdad sino puede que esté haciendo algo mal, cierto?

  • @RicardoLargaespada
    @RicardoLargaespada หลายเดือนก่อน +3

    ¿Qué olimpiada será?

    • @orielcedeno9800
      @orielcedeno9800 หลายเดือนก่อน +3

      Claramente los juegos olimpicos de Los Angeles 1984😂😂

    • @IchigooMatematicas
      @IchigooMatematicas  หลายเดือนก่อน +1

      Hola, interesante respuesta 😌.

    • @IchigooMatematicas
      @IchigooMatematicas  หลายเดือนก่อน +2

      Hola, es de una olimpiada de Australia.

  • @marinocg1490
    @marinocg1490 หลายเดือนก่อน +5

    Bien, pero por qué canta cuando habla......?

    • @IchigooMatematicas
      @IchigooMatematicas  หลายเดือนก่อน +3

      Hola, muchas gracias por su apoyo y comentario.
      Es mi acento y forma de hablar.

  • @GUTY1729
    @GUTY1729 หลายเดือนก่อน +4

    Esa solución es única o hay más en R?

    • @IchigooMatematicas
      @IchigooMatematicas  หลายเดือนก่อน +1

      Hola, en R es única.

    • @joryeo10
      @joryeo10 หลายเดือนก่อน +2

      Hay infinitas soluciones en los números reales pues es una ecuación con 2 incógnitas. En los números enteros sí es la única solución.

    • @GUTY1729
      @GUTY1729 หลายเดือนก่อน +1

      @@IchigooMatematicas alguna demostracion que es unica?

    • @hugoac0379
      @hugoac0379 หลายเดือนก่อน +1

      ​@@joryeo10 Hola, usted sabe porque tiene infinita soluciones en los reales, pero una sola solución en los enteros?

    • @GUTY1729
      @GUTY1729 หลายเดือนก่อน +1

      @@hugoac0379 por ahí hay algún programa en 3D que haga la intersección del plano con ese solido. Para mí también hay infinitas soluciones.

  • @MrSISRUSSELL
    @MrSISRUSSELL หลายเดือนก่อน +2

    Fácil

  • @rafaelbarranco9654
    @rafaelbarranco9654 หลายเดือนก่อน +3

    Te faltó poner con k distinto de 0 ha

  • @mugiwaranoluffy5336
    @mugiwaranoluffy5336 หลายเดือนก่อน +2

    2¹¹-2⁶

  • @juancarlosperafan2749
    @juancarlosperafan2749 หลายเดือนก่อน +3

    m=10 n=6

    • @elumath73
      @elumath73 หลายเดือนก่อน +3

      m = 11

  • @daviddambrosio1426
    @daviddambrosio1426 หลายเดือนก่อน +6

    Fue muy enroscada la solución. Si directamente transformas el 1984 en 2048(potencia de 2) - 64(potencia de 2), te queda: 2^m - 2^n = 2^11 - 2 ^6.
    Y listo ya está